Sum of Series with Prime Factors 2 and 3 | Reciprocal Positive Integers

  • Thread starter DivGradCurl
  • Start date
  • Tags
    Series
In summary: This is equal to((1+1/3+1/9+...)*S)=(3/2)*SBut we also know from the first part of the question that the sum of all the powers of (1/2) is equal to 1. So S=1so the original sum=(3/2)*S=(3/2)In summary, the sum of the series 1 + (1/2) + (1/3) + (1/4) + (1/6) + (1/8) + (1/9) + (1/12) + ... where the terms are the reciprocals of the positive integers whose only prime
  • #1
DivGradCurl
372
0
Find the sum of the series

[tex] 1 + \frac{1}{2} + \frac{1}{3} + \frac{1}{4} + \frac{1}{6} + \frac{1}{8} + \frac{1}{9} + \frac{1}{12} + \dotsb [/tex]

where the terms are the reciprocals of the positive integers whose only prime factors are 2s and 3s.

Well, here is my guess:

[tex] 1 + \frac{1}{2} + \frac{1}{3} + \frac{1}{4} + \frac{1}{6} + \frac{1}{8} + \frac{1}{9} + \frac{1}{12} + \dotsb = 1 + \sum _{n=1 } ^{\infty} \left( \frac{1}{2} \right) ^n + \sum _{n=1 } ^{\infty} \left( \frac{1}{3} \right) ^n + \sum _{n=1 } ^{\infty} \left( \frac{1}{2\cdot 3} \right) ^n + \mbox{ ? } = \sum \frac{1}{2^x \cdot 3 ^y} [/tex]

As you can readily observe, I'm really stuck. Maybe someone could give me a tip. Any help is highly appreciated.

Thanks
 
Physics news on Phys.org
  • #2
Look at :

[tex]1 + \sum _{n=1 } ^{\infty} \frac{1}{2^n} + \sum _{n=1 } ^{\infty} \frac{1}{3^n} + \left( \sum _{n=1 } ^{\infty} \frac{1}{2^n} \right) \cdot \left( \sum _{n=1 } ^{\infty} \frac{1}{3^n} \right)[/tex]
 
Last edited:
  • #3
From the final sum you got, it looks like you're on the right track (though the stuff in the middle looks wrong). It's actually pretty easy to evaluate; the method might be easier to see if you didn't use shorthand notation.
 
  • #4
Let S be the sum of powers of 1/2. Then besides S you will also get S/3, S/3^2, etc. Add them all together and you get...?
 
  • #5
krab said:
Let S be the sum of powers of 1/2. Then besides S you will also get S/3, S/3^2, etc. Add them all together and you get...?

This is much nicer !
 
  • #6
[tex] 1 + \frac{1}{2} + \frac{1}{3} + \frac{1}{4} + \frac{1}{6} + \frac{1}{8} + \frac{1}{9} + \frac{1}{12} + \dotsb [/tex]

(1) Let S be the sum of powers of 1/2.

[tex] S = \sum _{n=1} ^{\infty} \left( \frac{1}{2} \right) ^n = \frac{1}{2} + \frac{1}{4} + \frac{1}{8} + \dotsb = \frac{1}{1-\frac{1}{2}} - 1 = 1 [/tex]

(2) Then besides S you will also get S/3, S/3^2, etc.

[tex] 1 + \frac{1}{2} + \frac{1}{3} + \frac{1}{4} + \frac{1}{6} + \frac{1}{8} + \frac{1}{9} + \frac{1}{12} + \dotsb - S = 1 + \frac{1}{3} + \frac{1}{6} + \frac{1}{9} + \frac{1}{12} + \dotsb = S + \frac{S}{3} + \frac{S}{6} + \frac{S}{9} + \frac{S}{12} + \dotsb [/tex]

(3) Add them all together and you get...?

If I followed your directions correctly, I should be able to add all the numbers. First, I sum the series where the terms are the reciprocals of the positive integers whose only prime factors are 3s:

[tex] S + \frac{S}{3} + \frac{S}{6} + \frac{S}{9} + \frac{S}{12} + \dotsb = S \sum _{n=0} ^{\infty} \left( \frac{1}{3} \right) ^n \underbrace{ + \frac{S}{6} + \frac{S}{12} + \dotsb} _{OUTSIDE} [/tex]

But, as you can see, there are some terms left out of the series above. However, it seems to be implied on (2) it is the only one required to do the job ("you will also get S/3, S/3^2, etc"). According to Gokul43201, your method works fine, so I must have done quite a few mistakes up to this point. Could you please clarify that?

Thanks
 
Last edited:
  • #7
You could do a follow-up on Gokul's initial method and show that your sum may be written as:
[tex]S=(\sum_{i=0}^{\infty}\frac{1}{2^{i}})(\sum_{j=0}^{\infty}\frac{1}{3^{j}})[/tex]

(Note that this is just what krab implied..)
 
  • #8
It makes sense. So, we get

[tex] 1 + \frac{1}{2} + \frac{1}{3} + \frac{1}{4} + \frac{1}{6} + \frac{1}{8} + \frac{1}{9} + \frac{1}{12} + \dotsb = (\sum_{i=0}^{\infty}\frac{1}{2^{i}})(\sum_{j=0}^ {\infty}\frac{1}{3^{j}}) [/tex]

Thank you
 
  • #9
Correct.
I see I used "S" in a different sense than krab did, but fortunately, you got what I meant.
Note that Gokul's initial result is found through:
[tex](\sum_{i=0}^{\infty}\frac{1}{2^{i}})(\sum_{j=0}^{\infty}\frac{1}{2^{j}})=(1+\sum_{i=1}^{\infty}\frac{1}{2^{i}})(1+\sum_{j=1}^{\infty}\frac{1}{2^{j}})[/tex]
and then expanding the brackets.
 
  • #10
thiago_j said:
If I followed your directions correctly, I should be able to add all the numbers. First, I sum the series where the terms are the reciprocals of the positive integers whose only prime factors are 3s:

[tex] S + \frac{S}{3} + \frac{S}{6} + \frac{S}{9} + \frac{S}{12} + \dotsb = S \sum _{n=0} ^{\infty} \left( \frac{1}{3} \right) ^n \underbrace{ + \frac{S}{6} + \frac{S}{12} + \dotsb} _{OUTSIDE} [/tex]

But, as you can see, there are some terms left out of the series above. However, it seems to be implied on (2) it is the only one required to do the job ("you will also get S/3, S/3^2, etc"). According to Gokul43201, your method works fine, so I must have done quite a few mistakes up to this point. Could you please clarify that?

Thanks

To get all the terms in your series, you need every power of (1/2) multiplied by every power of (1/3). This follows from the definition of your series... So one way to get all the terms is to take all the different powers of 1/2, multipling by (1/3)^0, then adding all the powers of 1/2 multiplied by (1/3)^1, then adding all the powers of 1/2 multplied by (1/3)^2 etc... In this manner you get every power of (1/2) multiplied by every power of (1/3)

So your sum is:
(1+1/2 + 1/3 + 1/4 + 1/6 + 1/8 +1/9 +1/12 + ...)
= 1*(1+1/2 + 1/4 + 1/8 + 1/16 +...) + (1/3)(1+1/2 + 1/4 + 1/8 + 1/16 +...) + (1/9)(1+1/2 + 1/4 + 1/8 + 1/16 +...) +...

S=(1+1/2+1/4+1/8+...)

Substitue in S to the original sum, so the original sum=
1*S + (1/3)*S + (1/9)*S +...
 

What is the sum of the series with prime factors 2 and 3?

The sum of the series with prime factors 2 and 3 is an infinite series that includes all positive integers that can be expressed as a product of powers of 2 and 3.

How do you calculate the sum of the series with prime factors 2 and 3?

The sum of the series with prime factors 2 and 3 can be calculated using the formula S = (2^n - 1)(3^n - 1)/(2-3), where n is the number of terms in the series.

What are the prime factors of the numbers in the series with prime factors 2 and 3?

The prime factors of the numbers in the series with prime factors 2 and 3 are only 2 and 3, as these are the only prime numbers that can be used to form the terms in the series.

What is the pattern of the series with prime factors 2 and 3?

The pattern of the series with prime factors 2 and 3 is that each term is formed by multiplying a power of 2 with a power of 3, and the terms are arranged in increasing order.

What is the significance of the series with prime factors 2 and 3 in mathematics?

The series with prime factors 2 and 3 has significance in number theory and is used in various mathematical proofs and calculations. It also has applications in computer science and cryptography.

Similar threads

  • Introductory Physics Homework Help
Replies
17
Views
398
  • Introductory Physics Homework Help
3
Replies
95
Views
4K
  • Introductory Physics Homework Help
Replies
23
Views
345
  • Introductory Physics Homework Help
Replies
2
Views
232
  • Introductory Physics Homework Help
Replies
12
Views
731
  • Introductory Physics Homework Help
Replies
7
Views
817
  • Introductory Physics Homework Help
Replies
1
Views
901
  • Introductory Physics Homework Help
Replies
3
Views
144
  • Introductory Physics Homework Help
Replies
15
Views
292
  • Introductory Physics Homework Help
Replies
2
Views
304
Back
Top